Difference between revisions of "1998 AHSME Problems/Problem 29"

(Solution 1)
Line 6: Line 6:
  
 
== Solution 1 ==
 
== Solution 1 ==
Sadly, I don't actually have a solution. However, after doing some work on Geogebra, I have convinced myself that the answer is almost certainly A
+
The answer is actually (D)
 
 
  
 
== Solution 2 ==
 
== Solution 2 ==

Revision as of 22:28, 7 December 2014

Problem

A point $(x,y)$ in the plane is called a lattice point if both $x$ and $y$ are integers. The area of the largest square that contains exactly three lattice points in its interior is closest to

$\mathrm{(A) \ } 4.0 \qquad \mathrm{(B) \ } 4.2 \qquad \mathrm{(C) \ } 4.5 \qquad \mathrm{(D) \ } 5.0 \qquad \mathrm{(E) \ }  5.6$


Solution 1

The answer is actually (D)

Solution 2

Apply Pick's Theorem. 4 lattice points on the border edges, 3 points in the interior. $A = I + \frac{B}{2} -1$, implying that $max(A) = 4$, $\boxed{A}$

See also

1998 AHSME (ProblemsAnswer KeyResources)
Preceded by
Problem 28
Followed by
Problem 30
1 2 3 4 5 6 7 8 9 10 11 12 13 14 15 16 17 18 19 20 21 22 23 24 25 26 27 28 29 30
All AHSME Problems and Solutions

The problems on this page are copyrighted by the Mathematical Association of America's American Mathematics Competitions. AMC logo.png